Sie sind auf Seite 1von 5

MAT1203 Algebra Lineal

Ayudant a 8
Formas cuadr aticas y Determinante
Jorge Faundes

1. Sea p(x) = ax2 + bx + c. Muestre que si a > 0 y b2 4ac < 0 entonces p(x) > 0 x R. 2. Sea A matriz antisim etrica. Muestre que xT Ax = 0 x Rn y que I A es invertible. 3. Existe una matriz cuadrada A con elementos reales tal que A + zI sea invertible para todo complejo z ? Encuentre una matriz cuadrada A tal que A + xI sea invertible para todo x real. Demuestre que una matriz que cumpla la condici on anterior debe ser de tama no n n para n par. 4. Sea 1 1 0 A = 1 2 0 1 Clasique la forma cuadr atica xT Ax seg un el valor de R y usando una sustituci on adecuada expr esela como suma de cuadrados ponderados. 5. Sea Tn matriz tridiagonal de unos de n n. Encuentre una f ormula recursiva para det(Tn ) y u sela para encontrar det(T4 ). 6. Sean A, B matrices cuadradas invertibles. a ) Muestre que Adj (AT ) = (Adj (A))T . b ) Encuentre Adj (Adj (A)) en funci on de A. c ) Demuestre que Adj (AB ) = Adj (B )Adj (A). 7. Decida si las siguientes armaciones son verdaderas o falsas. a ) Sea A matriz antisim etrica, entonces det(A) = 0. b ) Si Q es una matriz cuadrada tal que QT Q = I entonces |det(Q)| = 1. c ) Para A, B, C matrices cuadradas del mismo tama no sea M= A B O C

Entonces det(M ) = det(A)det(C ).

8. Sea 1 1 0 0 x h 1 0 A3 = x2 hx h 1 x3 hx2 hx h Demuestre que det(A3 ) = (x + h)3 . Propuesto: Dena la matriz correspondiente An y demuestre que det(An ) = (x + h)n n N.

Soluciones:
1. Consideremos la forma cuadr atica p(x, y ) = ax2 + bxy + cy 2 . Notamos que p(x, 1) = p(x) y adem as que la matriz sim etrica que representa a la forma cuadr atica es A= a b/2 b/2 c

Como det(A1 ) = a > 0 y det(A) = ac b2 /4 > 0 (por la desigualdad del enunciado) sigue que la forma cuadr atica es positiva denida, es decir, p(x, 1) = p(x) > 0 x R. 3. Para la primera parte, dada una matriz cuadrada A, llamemos P (z ) = det(A + zI ). Es claro que P (z ) es un polinomio, y por lo tanto tiene al menos una ra z en C (por el Teorema Fundamental del Algebra), es decir, existe z C tal que det(A + z I ) = 0. Luego, la matriz A + z I no es invertible, por lo que no existe ninguna matriz que cumpla lo pedido en el enunciado. Ahora, busquemos una matriz A tal que A + xI sea invertible para todo x real. De forma an aloga al caso anterior, si A es de n n se tiene que Q(x) = det(A + xI ) es un polinomio de grado n. Luego, si n es impar se tiene que el polinomio tiene al menos una ra z real (recordar que un polinomio de coecientes reales s olo tiene un n umero par de soluciones complejas), de donde existe x R tal que det(A + x I ) = 0, es decir, A + x I no es invertible. Luego, si A es de n n tal que A + xI sea invertible para todo x real se debe tener n par. Por lo tanto, busquemos una matriz de 2 2 que cumpla lo pedido. a b , entonces Sea A = c d a+x b c d+x Buscamos que det(A + xI ) = 0 para todo x real. Lo anterior es equivalente a que el polinomio r(x) = det(A + xI ) = (a + x)(d + x) bc = x2 + (a + d)x + ad bc no tenga ra ces reales, es decir, que tenga discriminante negativo. Luego, se requiere (a + d)2 4(ad bc) < 0. 1 1 Por ejemplo, se puede tomar la matriz A = . 1 1 A + xI =

5. Para calcular un determinante de forma recursiva, debemos mirar la matriz de manera recursiva. Para n > 2 notamos que 0 0 0 . . . . Tn2 . . . . Tn1 . = Tn = . 1 0 1 0 1 1 1 0 1 1 0 0 1 1 Luego, si desarrollamos el determinante por cofactores en la u ltima columna se obtiene ) det(Tn ) = (1)2n det(Tn1 ) + (1)2n1 det(Tn 2 donde 0 . . Tn2 . Tn = 2 1 0 0 1
Si calculamos el determinante de Tn ltima la, vemos que 2 por cofactores en la u 2 n 2 det(Tn2 ) = (1) det(Tn2 ). As , det(Tn ) = det(Tn1 ) det(Tn2 ) n > 2. Para calcular det(T4 ), notamos que det(T1 ) = 1, det(T2 ) = 0 y as det(T3 ) = 0 1 = 1. Finalmente, det(T4 ) = 1 0 = 1.

7.

a ) Esta armaci on es falsa. Basta tomar A =

0 1 que es una matriz antisim etrica 1 0 de determinante no nulo. En la ayudant a vimos que la armaci on es cierta si n es impar, lo que es f acil de demostrar tomando determinante en la igualdad A = AT .

b ) Aplicando determinante en QT Q = I obtenemos det(QT )det(Q) = det(I ) = det(Q)2 = 1. Luego la armaci on es verdadera. c ) Esta armaci on es verdadera. Para probarla, basta notar que si separamos la matriz en los 2 siguientes bloques: M1 M1 = (A B ), M2 = (O C ) entonces M = . M2 Ahora, podemos escalonar M1 hasta dejar A en su forma escalonada, es decir, M1 (esc(A) B ) donde B es la matriz que resulta al aplicarle a B las operaciones elementales que se usaron para escalonar A. De la misma forma, escalonamos M2 hasta dejarla de la forma M2 (O esc(C )). As , M esc(M ) = esc(A) B O esc(C ) .

Notamos que la matriz anterior es triangular y as , a c c c . det(M ) = (1)r+s a a n n 1 2 1 2 4

Donde r y s son la cantidad de intercambio de las que se usaron para escalonar M1 y M2 respectivamente. Adem as, a i son los elementos de la diagonal de esc(A) y ci son los elementos de la diagonal de esc(C ). s Finalmente, como det(A) = (1)r a 1 a2 an y det(C ) = (1) c1 c2 cn entonces se tiene det(M ) = det(A)det(C ) que es lo que se buscaba probar.

Das könnte Ihnen auch gefallen